Difference between revisions of "2016 AMC 12B Problems/Problem 16"

(Solution)
(Solution)
Line 22: Line 22:
  
 
We have found all 7 solutions and our answer is <math>\boxed{\textbf{(E)} \, 7}</math>.
 
We have found all 7 solutions and our answer is <math>\boxed{\textbf{(E)} \, 7}</math>.
 +
 +
==Solution 2==
 +
 +
The sum from <math>a</math> to <math>b</math> where <math>a</math> and <math>b</math> are integers and <math>a>b </math> is <math>\dfrac{(a-b+1)(a+b)}{2}</math>
 +
 +
<math>(a-b+1)(a+b)=2\cdot 3\cdot 5\cdot 23</math>
 +
 +
If we factor 345 into all of its factor groups (exg (5,69) or (15,23)) we will have <math>(c,d)</math> where <math>c<d</math>.
 +
 +
The number of possible values for <math>c</math> is half the number of factors of 345 which is <math>\frac{1}{2}\cdot2\cdot2\cdot2\cdot2=8</math>
 +
 +
However, we have one extraneous case of <math>(1,345)</math> because here, <math>c=d</math> and we have the sum of one consecutive number which is nt allowed by the question.
 +
 +
Thus the answer is <math>8-1=7</math>
 +
 +
<math>\boxed{\textbf{(E)} \, 7}</math>.
  
 
==See Also==
 
==See Also==
 
{{AMC12 box|year=2016|ab=B|num-b=15|num-a=17}}
 
{{AMC12 box|year=2016|ab=B|num-b=15|num-a=17}}
 
{{MAA Notice}}
 
{{MAA Notice}}

Revision as of 02:50, 22 February 2016

Problem

In how many ways can $345$ be written as the sum of an increasing sequence of two or more consecutive positive integers?

$\textbf{(A)}\ 1\qquad\textbf{(B)}\ 3\qquad\textbf{(C)}\ 5\qquad\textbf{(D)}\ 6\qquad\textbf{(E)}\ 7$

Solution

We proceed with this problem by considering two cases, when: 1) There are an odd number of consecutive numbers, 2) There are an even number of consecutive numbers.

For the first case, we can cleverly choose the convenient form of our sequence to be \[a-n,\cdots, a-1, a, a+1, \cdots, a+n\]

because then our sum will just be $(2n+1)a$. We now have \[(2n+1)a = 345\] and $a$ will have a solution when $\frac{345}{2n+1}$ is an integer, namely when $2n+1$ is a divisor of 345. We check that \[2n+1 = 3, 5, 15, 23\] work, and no more, because $2n+1=1$ does not satisfy the requirements of two or more consecutive integers, and when $2n+1$ equals the next biggest factor, $69$, there must be negative integers in the sequence. Our solutions are $\{114,115, 116\}, \{67, \cdots, 71\}, \{16, \cdots, 30\}, \{4, \cdots, 26\}$.

For the even cases, we choose our sequence to be of the form: \[a-(n-1), \cdots, a, a+1, \cdots, a+n\] so the sum is $\frac{(2n)(2a+1)}{2} = n(2a+1)$. In this case, we find our solutions to be $\{172, 173\}, \{55,\cdots, 60\}, \{30,\cdots, 39\}$.

We have found all 7 solutions and our answer is $\boxed{\textbf{(E)} \, 7}$.

Solution 2

The sum from $a$ to $b$ where $a$ and $b$ are integers and $a>b$ is $\dfrac{(a-b+1)(a+b)}{2}$

$(a-b+1)(a+b)=2\cdot 3\cdot 5\cdot 23$

If we factor 345 into all of its factor groups (exg (5,69) or (15,23)) we will have $(c,d)$ where $c<d$.

The number of possible values for $c$ is half the number of factors of 345 which is $\frac{1}{2}\cdot2\cdot2\cdot2\cdot2=8$

However, we have one extraneous case of $(1,345)$ because here, $c=d$ and we have the sum of one consecutive number which is nt allowed by the question.

Thus the answer is $8-1=7$

$\boxed{\textbf{(E)} \, 7}$.

See Also

2016 AMC 12B (ProblemsAnswer KeyResources)
Preceded by
Problem 15
Followed by
Problem 17
1 2 3 4 5 6 7 8 9 10 11 12 13 14 15 16 17 18 19 20 21 22 23 24 25
All AMC 12 Problems and Solutions

The problems on this page are copyrighted by the Mathematical Association of America's American Mathematics Competitions. AMC logo.png